Question and Answers Forum

All Questions      Topic List

Logarithms Questions

Previous in All Question      Next in All Question      

Previous in Logarithms      Next in Logarithms      

Question Number 116491 by bemath last updated on 04/Oct/20

(0.16)^(log _(2.5) ((1/3)+(1/3^2 )+(1/3^3 )+...))  =?

$$\left(\mathrm{0}.\mathrm{16}\right)^{\mathrm{log}\:_{\mathrm{2}.\mathrm{5}} \left(\frac{\mathrm{1}}{\mathrm{3}}+\frac{\mathrm{1}}{\mathrm{3}^{\mathrm{2}} }+\frac{\mathrm{1}}{\mathrm{3}^{\mathrm{3}} }+...\right)} \:=? \\ $$

Answered by bobhans last updated on 04/Oct/20

Only applying property of logarithm  ⇒ a^(log _a  f(x))  = f(x)  ⇒((2/5))^(2.log _(2.5) ((1/3)+(1/3^2 )+(1/3^3 )+...)) = ((1/3)+(1/3^2 )+(1/3^3 )+...)^(−2)       = (((1/3)/(1−(1/3))))^(−2) = ((1/2))^(−2)  = (2)^2  = 4

$$\mathrm{Only}\:\mathrm{applying}\:\mathrm{property}\:\mathrm{of}\:\mathrm{logarithm} \\ $$$$\Rightarrow\:\mathrm{a}^{\mathrm{log}\:_{\mathrm{a}} \:\mathrm{f}\left(\mathrm{x}\right)} \:=\:\mathrm{f}\left(\mathrm{x}\right) \\ $$$$\Rightarrow\left(\frac{\mathrm{2}}{\mathrm{5}}\right)^{\mathrm{2}.\mathrm{log}\:_{\mathrm{2}.\mathrm{5}} \left(\frac{\mathrm{1}}{\mathrm{3}}+\frac{\mathrm{1}}{\mathrm{3}^{\mathrm{2}} }+\frac{\mathrm{1}}{\mathrm{3}^{\mathrm{3}} }+...\right)} =\:\left(\frac{\mathrm{1}}{\mathrm{3}}+\frac{\mathrm{1}}{\mathrm{3}^{\mathrm{2}} }+\frac{\mathrm{1}}{\mathrm{3}^{\mathrm{3}} }+...\right)^{−\mathrm{2}} \\ $$$$\:\:\:\:=\:\left(\frac{\frac{\mathrm{1}}{\mathrm{3}}}{\mathrm{1}−\frac{\mathrm{1}}{\mathrm{3}}}\right)^{−\mathrm{2}} =\:\left(\frac{\mathrm{1}}{\mathrm{2}}\right)^{−\mathrm{2}} \:=\:\left(\mathrm{2}\right)^{\mathrm{2}} \:=\:\mathrm{4} \\ $$

Terms of Service

Privacy Policy

Contact: info@tinkutara.com